Anatomy 3

You might also like

Download as pdf
Download as pdf
You are on page 1of 394
° Question 407 of 611 v F ° What is the correct embryological origin of the stapes? First pharyngeal arch Third pharyngeal arch Fourth pharyngeal arch Fifth pharyngeal arch Embryological origin stapes = 2nd pharyngeal arch. ‘The ectoderm covering the outer aspect of the second arch originates from a strip of ‘ectoderm lateral to the metencephalic neural fold. The cartilaginous element to this, ‘eponymously known as Reicherts cartilage extends from the otic capsule to the midline on each side. Its dorsal end separates and becomes enclosed in the tympanic cavity as the stapes. The dorsal ends of the cartilages of the first and second pharyngeal arches articulate superior to the tubotympanic recess. These cartilages form the malleus, incus and stapes. At least part of the malleus is formed from the first arch and the stapes from the second arch. The incus is most likely to arise from the first arch. Pharyngeal arches * These develop during the fourth week of embryonic growth from a series of mesodermal outpouchings of the developing pharynx. They develop and fuse in the ventral midline. Pharyngeal pouches form on the endodermal side between the arches There are 6 pharyngeal arches, the fifth does not contribute any useful structures and often fuses with the sixth arch Pharyngeal arches Pharyngeal Muscular Skeletal arch contributions contributions Endocrine Artery Nerve First Muscles of Maxilla na Maxillary Mandibular mastication Meckels Extemal Anterior belly of cartilage carotid digastric Incus Mylohyoid Tensor tympanic Tensor vel palatini Second Buccinator Platysma Muscles of facial expression Stylohyoid Posterior belly of digastric Stapedius Third Stylopharyngeus Fourth Cricothyroid Allintrinsic muscles of the soft palate sixth Allintrinsic muscles of the larynx (except cricothyroid) Bie &- ‘Save my notes arch ‘Search textbook. Malleus Stapes n/a styloid process Lesser hom and upper body of hyoid Greater horn Thymus and lower Inferior part of hyoid —_parathyroids Thyroid and Superior epigiottic parathyroids cartilages cricoid, n/a arytenoid and comiculate cartilages Inferior branch of superior thyroid artery Stapedial artery Common and internal carotid Right- subclavian artery, Left-aortic arch Right - Pulmonary artery, Left: Pulmonary artery and ductus arteriosus Facial Glossophary Vagus Vagus and recurrent laryngeal net °o Question 402 of 611 Pa Bp e A baby is found to have a Klumpke's palsy post delivery. Which of the following Is most likely to be present? Weak elbow flexion Pronated forearm Adducted shoulder Shoulder medially rotated S Features of Klumpkes Paralysis ‘+ Claw hand (MCP joints extended and IP joints flexed) ‘+ Loss of sensation over medial aspect of forearm and hand. + Horner's syndrome '* Loss of flexors of the wrist ACB, T1 root lesion is called Klumpke's paralysis and is caused by delivery with the arm extended. (« "9 | improve Brachial plexus * The brachial plexus extends from the neck to the axilla. It is formed by the ventral rami of the fifth to the eighth cervical nerves with the ascending part of the first thoracic nerve. Location of the plexus The ventral rami which form the plexus enter the lower part of the posterior triangle of the neck in series with the ventral rami of the cervical plexus. The second part of the subclavian artery lies immediately anterior to the lower two rami. The upper three rami intermingle and pass inferolaterally towards the axilla and subclavian artery. They are enclosed within an extension of the prevertebral fascia. In the neck the plexus lies deep to platysma, the supraclavicular nerves, inferior belly of omohyoid and the transverse cervical artery. It then passes deep to the clavicle and the suprascapular vessels, to enter the axilla, and thence surround the second part of the axillary artery Composition of the plexus Ventral rami, the roots of the plexus, lie between scalenus medius and anterior. As they enter the posterior triangle, the upper two (C5.6) and lower two (C8, T1) roots of the plexus unite to form the upper and lower trunks of the plexus respectively. Meanwhile, C7 continues as the middle trunk. The lower trunk may groove the superior surface of the first rib posterior to the subclavian artery, and the root from the first ventral ramus is always in contact with it Each trunk divides into ventral and dorsal divisions which are destined to supply the anterior (flexor) and posterior (extensor) parts of the upper limb. The cords of the plexus are formed in the axilla. The dorsal divisions unite to form the posterior cord (C5-8). The ventral divisions of the upper and middie trunks unite to form the lateral cord (C5-7), while the ventral divisions of the lower trunk continues as the medial cord (C8-T1). The cords are named according to their relationship to the axillary artery. Each cord terminates by dividing into two main branches at the beginning of the third part of the artery. ‘Sympathetic communications The fifth and sixth cervical ventral rami receive grey rami communicantes from the middle cervical ganglion, while the two or more grey rami communicantes pass from the inferior cervical ganglion to the seventh and eighth cervical ventral rami. The first thoracic ventral ramus receives Its grey ramus from the cervicothoracie ganglion. Its for this reason that inferior plexus injury can be complicated by a Horners syndrome. Summary origi Anterior rami of C5 to T1 Sections of the + Roots, trunks, divisions, cords, branches plexus + Mnemonic:Real Teenagers Drink Cold Beer Roots + Located in the posterior triangle * Pass between scalenus anterior and medius. Trunks * Located posterior to middle third of clavicle * Upper and middle trunks related superiorly to the subclavian artery * Lower trunk passes over 1st rib posterior to the subclavian artery Divisions Apex of axilla Cords Related to axillary artery Diagram illustrating the branches of the brachial plexus Diagram illustrating the branches of the brachial plexus Cutaneous sensation of the upper limb BlaW- S22) & go | Save my notes ° Question 403 of 611 v B © ‘The vertebral artery traverses all of the following except? Transverse process of C6 @ Transverse process of the axis Vertebral canal Foramen magnum, ‘The vertebral artery passes through the foramina which are located in the transverse processes of the cervical vertebra, It does not traverse the intervertebral foramen. [oa | | improve | Vertebral artery * ‘The vertebral artery is the first branch of the subclavian artery. Anatomically it is divisible into 4 regions: ‘+ The first part runs to the foramen in the transverse process of C6. Anterior to this part lies the vertebral and internal jugular veins. On the left side the thoracic duct is also an anterior relation, ‘+ The second part runs superiorly through the foramina of the the transverse processes of the upper 6 cervical vertebrae. Once it has passed through the transverse process of the axis it then turns superolaterally to the atlas. It is accompanied by a venous plexus and the inferior cervical sympathetic ganglion. ‘+ The third part runs posteromedially on the lateral mass of the atlas. It enters the sub occipital triangle, in the groove of the upper surface of the posterior arch of the atlas. it then passes anterior to the edge of the posterior atlanto-occipital membrane to enter the vertebral canal ‘+ The fourth part passes through the spinal dura and arachnoid, running superiorly and anteriorly at the lateral aspect of the medulla oblongata. At the lower border of the pons it unites to form the basilar artery. VY Question 404 of 611 - Pe wy ‘A22 year old man undergoes a superficial parotidectomy for a pleomorphic adenoma. The operation does not proceed well and a diathermy malfunction results in division of the buccal branch of the facial nerve. Which of the following muscles will not demonstrate impaired function as a result? Zygomaticus minor = = Buccinator Levator angull oris eo Risorius oe Buccal branch supplies Zygomaticus minor Elevates upper lip Risorius Aids smile Buccinator Pulls corner of mouth backward and compresses cheek Levator angull oris Pulls angles of mouth upward and toward midline Orbicularis Closes and tightens lips together Nasalis Flares nostrils and compresses nostrils « Improve Facial nerve * ‘The facial nerve is the main nerve supplying the structures of the second embryonic branchial arch. It is predominantly an efferent nerve to the muscles of facial expression, digastric muscle and also to many glandular structures. It contains a few afferent fibres which originate in the cells of its genicular ganglion and are concerned with taste. ‘Supply - ‘face, ear, taste, tear’ * Face: muscles of facial expression * Ear: nerve to stapedius + Taste: supplies anterior two-thirds of tongue * Tear: parasympathetic fibres to lacrimal glands, also salivary glands Path Subarachnoid path * Origin: motor- pons, sensory- nervus intermedius * Pass through the petrous temporal bone into the internal auditory meatus with the vestibulocochlear nerve. Here they combine to become the facial nerve. canal path + The canal passes superior to the vestibule of the inner ear + At the medial aspect of the middle ear, it becomes wider and contains the geniculate ganglion. - 3 branches: 1. greater petrosal nerve 2.nerve to stapedius 3. chorda tympani Stylomastoid foramen + Passes through the stylomastoid foramen (tympanic cavity anterior and mastoid antrum posteriorly) * Posterior auricular nerve and branch to posterior belly of digastric and stylohyoid muscle Face Enters parotid gland and divides into 5 branches: + Temporal branch + Zygomatic branch * Buccal branch + Marginal mandibular branch + Cervical branch Bol ee = = Tr & @ © [avery nots Search Search textbook Go Q Google search on "Facial nerve" Suggest tink Suggest media ° Question 405 of 611 x p °o A surgical resection specimen is analysed histologically. The pathologist comments that at the periphery of the resected specimen, oxyphil cells are identified. In which of the structures listed below are these cells typically found? Thymus Da id go ea Lymph node 2 ‘Adrenal gland eo Oxyphil cells are typically found In parathyroid glands | oe | imrove | , rs Parathyroid glands- anatomy * + Four parathyroid glands * Located posterior to the thyroid gland + They lie within the pretracheal fascia Embryology The parathyrolds develop from the extremities of the third and fourth pharyngeal pouches. The parathyroid derived from the fourth pharyngeal pouch are located more superiorly and are associated with the thyroid gland. Those derived from the third pharyngeal pouch lie more inferiorly and may become associated with the thymus. Blood supply ‘The blood supply to the parathyroid glands is derived from the inferior and superior thyroid arteriesf']. There is a rich anastomosis between the two vessels. Venous drainage is into the thyroid veins. Relations Laterally Common carotid Medially Recurrent laryngeal nerve, trachea Anterior Thyroid Posterior Pretracheal fascia References ‘Nobori, M, et al, Blood supply of the parathyroid gland from the superior thyrold artery Surgery, 1994. 115(4): p. 417-23. eee ge [seve my ote Search ‘Search textbook ‘Q Google search on “Parathyroid glands- anatomy" Suggest nk suggest media Dashboard eyoaarona u 12 13 14 WOT ATRER: <) 47Ci ca RC < << o Question 406 of 611 v B oO ‘A.43 year old lady Is undergoing a total thyroidectomy for an extremely large goltre. The surgeons decide that access may be improved by division of the infra hyoid strap muscles. At which of the following sites should they be divided? En - In their lower half @ Inthe middle @ At their origin from the hyoid At the point of their insertion z Should the strap muscles require division during surgery they should be divided in their upper half. This is because their nerve supply from the ansa cervicalis enters in their lower half 4 | 9 | improve | Anterior triangle of the neck * Boundaries Anterior border of the Sternocleidomastoid Lower border of mandible Anterior midline Sub triangles (divided by Digastric above and Omohyoid) ‘+ Muscular triangle: Neck strap muscles *+ Carotid triangle: Carotid sheath + Submandibular Triangle (digastric) Contents of the anterior triangle Digastric triangle Submandibular gland Submandibular nodes Facial vessels Hypoglossal nerve Musculartriangle Strap muscles Jugular vein Carotid triangle Ansa cervicalis, Nerve supply to digastric muscle * Anterior: Mylohyoid nerve * Posterior: Facial nerve Carotid sheath (Common carotid, vagus and internal jugular vein) Try gee ‘Save my notes Search Search textbook Q Google search on “Anterior triangle of the neck” 4 Suggest lnk Suggest media fj] @ Dashboard Rona sae ‘73 year old man Is due to undergo a radical prostatectomy for carcinoma of the prostate gland. To which of the following lymph nodes will the tumour drain primarily? Para aortic. iia - ‘Superficial inguinal eo Meso rectal eo None of the above @ ‘The prostate lymphatic drainage is primarily to the internal iliac nodes and also the sacral nodes. Although internal iliac is the first site Improve Prostate gland * ‘The prostate gland is approximately the shape and size of a walnut and Is located inferior to the bladder. It is separated from the rectum by Denonvilliers fascia and its blood supply is derived from the internal iliac vessels (via inferior vesical artery). The internal sphincter lies at the apex of the gland and may be damaged during prostatic surgery, affected individuals may complain of retrograde ejaculation. ‘Summary of prostate gland Arterial supply Inferior vesical artery (from internal iliac) Venous Prostatic venous plexus (to paravertebral veins) drainage Lymphatic Internal iliac nodes drainage Innervation Inferior hypogastric plexus + Transverse diameter (4m) * AP diameter (2cm) * Height (3em) Lobes *+ Posterior lobe: posterior to urethra + Median lobe: posterior to urethra, in between ejaculatory ducts * Lateral lobes x 2 * Isthmus Zones * Peripheral zone: subcapsular portion of posterior prostate. Most prostate cancers are here * Central zone * Transition zone * Stroma Relations Pubic symphysis Anterior Prostatic venous plexus Posterior Denonvilliers fascia, Rectum Ejaculatory ducts Lateral Venous plexus (lies on prostate) Levator ani (Immediately below the puboprostatic ligaments) x Bie @- a wo Save my notes ‘Search Search textbook. B Q Google search on "Prostate glanc” ° Question 408 of 611 x B o What is the lymphatic drainage of the female urethra? | spot ni @e Deep inguinal nodes @ External iliac nodes Para-aortic nodes eo ‘The entire female urethra drains to the internal iliac nodes, of | @ | Improve Urethral anatomy * Female urethra The female urethra is shorter and more acutely angulated than the male urethra. Itis an extra- peritoneal structure and embedded in the endopelvic fascia, The neck of the bladder is subjected to transmitted intra-abdominal pressure and therefore deficiency in this area may result in stress urinary incontinence. Between the layers of the urogenital diaphragm the female urethra is, surrounded by the external urethral sphincter, this is innervated by the pudendal nerve. It Ultimately lies anterior to the vaginal orifice. Male urethra In males the urethra is much longer and Is divided into four parts. Pre-prostatic Extremely short and lies between the bladder and prostate gland. It has a urethra stellate lumen and is between 1 and 1.5cm long.innervated by sympathetic noradrenergic fibres, as this region is composed of striated muscles bundles they may contract and prevent retrograde ejaculation. Prostatic This segment is wider than the membranous urethra and contains several urethra ‘openings for the transmission of semen (at the midpoint of the urethral crest). Membranous _ Narrowest part of the urethra and surrounded by external sphincter. It urethra traverses the perineal membrane 2.5cm postero-Inferior to the symphysis pubis. Travels through the corpus spongiosum on the underside of the penis. Itis urethra the longest urethral segment.tt is dilated at its origin as the infrabulbar fossa and again in the glans penis as the navicular fossa. The bulbo-urethral ‘lands open into the spongiose section of the urethra 2.5em below the perineal membrane. The urothelium is transitional in nature near to the bladder and becomes squamous more distally. ‘Save my notes Search Search textbook B ‘Q Google search on Urethral anatomy" Suggest lnk suggest media Dashboard e@Voursona " 12. 13 14 15 16 7 Winer tUTREG Ry4110F40 6026 Geleeiec re ° auestion 409 of 611 * B ° Which of the structures listed below are most closely related to the axillary nerve within the quadrangular space? Axillary artery Anterior circumflex humeral vessels [is artery Acromiothoracic artery ‘The posterior circumflex humeral vessels which are branches of the axillary artery are related to the axillary nerve within the quadrangular space. "9 | improve Axillary nerve * * Terminal branch of the posterior cord of the brachial plexus + Root values C5 and C6 + Descends posterior to the axillary artery at the lower border of subscapularis and then passes through quadrangular space with the posterior circumflex humeral vessels *+ Divides into anterior and posterior branches * Innervates deltoid muscle and small patch of skin over deltoid 38 Question 410 of 611 v B ° ‘Aman sustains a laceration between the base of the little finger and wrist. Several weeks after the injury there is loss of thumb adduction power. Which nerve is most likely to have been injured? Superficial ulnar nerve Qa Median nerve eQ Radial nerve e eo Recurrent branch of median nerve Ulnar nerve injury at wrist * Branches of the ulnar nerve in the wrist and hand At the wrist the ulnar nerve divides into superficial and deep branches. The superficial branch lies deep to the palmaris brevis. It divides into two; to produce digital nerves, which innervate the skin of the medial third of the palm and the palmar surface of one and a half fingers. ‘The deep branch arises from the nerve on the flexor retinaculum lateral to the pisiform bone. It asses posteriorly between the abductor and short flexor of the little finger supplying them, and supplying and piercing the opponens digiti minimi near its origin from the flexor retinaculum, turns laterally over the distal surface of the Hook of the Hamate bone. It eventually passes between the two heads of adductor pollicis with the deep palmar arch and ends in the first dorsal interosseous muscle. In the palm the deep branch also innervates the lumbricals and interosseous muscles. Bis @-re Save my notes ° Question 417 of 611 x Bp ©} The thebesian veins contribute to the venous drainage of the heart. Into which of the following structures do they primarily drain? Superior vena cava Oblique vein Small cardiac vein The thebesian veins are numerous small veins running over the surface of the heart they drain into the heart itself. Usually this Is to the atrium directly. | | Improve Heart anatomy * The walls of each cardiac chamber comprise: + Epicardium = Myocardium + Endocardium Cardiac muscle is attached to the cardiac fibrous skeleton. Relations The heart and roots of the great vessels within the pericardial sac are related to the posterior aspect of the sternum, medial ends of the 3rd to Sth ribs on the left and their associated costal cartilages. The heart and pericardial sac are situated obliquely two thirds to the left and one third to the right of the median plane The pulmonary valve lies at the level of the left third costal cartilage. The mitral valve lies at the level of the fourth costal cartilage. Coronary sinus This lies in the posterior part of the coronary groove and receives blood from the cardiac veins. The great cardiac vein lies at its left and the middle and small cardiac veins lie on its right. The smallest cardiac vein (anterior cardiac vein) drains into the right atrium directly. Aortic sinus Right coronary artery arises from the right aortic sinus, the left is derived from the left aortic. sinus, which lies posteriorly. Features of the left ventricle as opposed to the right Structure Left Ventricle AV Valve ‘Mitral (double leaflet) Walls Twice as thick as right Trabeculae carnae ‘Much thicker and more numerous Right coronary artery ‘The RCA supplies: + Right atrium ‘+ Diaphragmatic part of the right ventricle * Usually the posterior third of the interventricular septum * The sino atrial node (60% cases) + The atrio ventricular node (80% cases) Left coronary artery ‘The LCA supplies: © Left atrium + Most of left ventricle + Part of the right ventricle * Anterior two thirds of the inter ventricular septum * The sino atrial node (remaining 40% cases) Innervation of the heart Autonomic nerve fibres from the superficial and deep cardiac plexus. These lie anterior to the bifurcation of the trachea, posterior to the ascending aorta and superior to the bifurcation of the pulmonary trunk. The parasympathetic supply to the heart is from presynaptic fibres of the vagus, nerves. Valves of the heart Mitral valve Aortic valve 2cusps 3 cusps First heart sound Second heart sound 1 anterior cusp anterior cusps Attached to chordae No chordae tendinae Pulmonary valve 3 cusps Second heart ‘sound anterior cusps No chordae Tricuspid valve 3 cusps First heart sound 2 anterior cusps Attached to chordae tendinae ° Question 412 of 611 v BR ©} ‘22 year old falls over and lands on a shard of glass. It penetrates the palmar aspect of his hand, immediately lateral to the pisiform bone. Which of the following structures is most likely to be injure? Palmar cutaneous branch of the median nerve = Lateral tendons of flexor digitorum superficialis oe Flexor carpi radialis tendons e Lateral tendons of flexor digitorum profundus e The ulnar nerve and artery are at most immediate risk in this injury. This Is illustrated in the image below: @ | Improve Hand * Hand * ‘Anatomy of the hand Bones + 8 Carpal bones + 5 Metacarpals + 14 phalanges 8 Interossel - Supplied by ulnar nerve + 4 palmar-adduct fingers + 4 dorsal- abduct fingers Lumbricals + Flex MCPJ and extend the IPJ. ‘© Origin deep flexor tendon and insertion dorsal extensor hood mechanism. * Innervation: 1st and 2nd- median nerve, 3rd and 4th- deep branch of the ulnar nerve. Thenar + Abduetor pollicis brevis eminence + Opponens pollicis + Flexor pollicis brevis, Hypothenar + Opponens digit! minimi eminence + Flexor digiti minimi brevis + Abductor digiti minimi Fasci 3nd compartments of the palm The fascia of the palm is continuous with the antebrachial fascia and the fascia of the dorsum of the hand. The palmar fascia Is thin over the thenar and hypothenar eminences. In contrast, the central palmar fascia is relatively thick. The palmar aponeurosis covers the soft tissues and overlies the flexor tendons. The apex of the palmar aponeurosis Is continuous with the flexor retinaculum and the palmaris longus tendon. Distally, it forms four longitudinal digital bands that attach to the bases of the proximal phalanges, blending with the fibrous digital sheaths. ‘A medial fibrous septum extends deeply from the medial border of the palmar aponeurosis to the 5th metacarpal. Lying medial to this are the hypothenar muscles. In a similar fashion, a lateral fibrous septum extends deeply from the lateral border of the palmar aponeurosis to the 3rd metacarpal. The thenar compartment lies lateral to this area. Lying between the thenar and hypothenar compartments is the central compartment. It contains the flexor tendons and their sheaths, the lumbricals, the superficial palmar arterial arch and the digital vessels and nerves. The deepest muscular plane is the adductor compartment, which contains adductor pollicis, Short muscles of the hand These comprise the lumbricals and interossel. The four slender lumbrical muscles flex the fingers at the metacarpophalangeal joints and extend the interphalangeal joint. The four dorsal interossei are located between the metacarpals and the three palmar interosse! lie on the palmar surface of the metacarpals in the interosseous compartment of the hand, Long flexor tendons and sheaths in the hand The tendons of FDS and FDP enter the common flexor sheath deep to the flexor retinaculum. The tendons enter the central compartment of the hand and fan out to their respective digital synovial sheaths. Near the base of the proximal phalanx, the tendon of FDS splits to permit the passage of FDP. The FDP tendons are attached to the margins of the anterior aspect of the base of the distal phalanx, The fibrous digital sheaths contain the flexor tendons and their synovial sheaths. These extend from the heads of the metacarpals to the base of the distal phalanges. Palmar Interossei Note that there are 4 palmar interossel. The first Is a small slip of muscle which arises from the Ulnar side of the base of the first metacarpal and passes between the head of the first dorsal Interosseous and the oblique head of adductor pollicis to insert into the ulnar base of the of the proximal phalanx of the thumb. The second arises from the ulnar side of the body of the second metacarpal and is inserted into the ulnar side of the extensor hood of the index. The third and fourth palmar interossel arise from the radial sides of the bodies of the 4th and Sth metacarpals respectively and insert into the radial sides of the extensor hoods of the ring and little fingers. Tr gy a © Save my notes 8 Question 413 of 611 v 5 Oo ‘A 45 year old man presents with bilateral inguinal hernias. The surgical team decide to repair these laparoscopically through an extraperitoneal approach. Through an infraumbilical incision the surgeons displace the inferior aspect of the rectus abdominis muscle anteriorly and place a prosthetic mesh into the area to repair the hernias. Which structure will lie posterior to the mesh? Internal oblique aponeurosis Posterior aspect of the rectus sheath @ Qo Extemal oblique aponeurosis, @ eo Bucks fascia During a TEP repair of inguinal hernia the only structure to lie posterior to the mesh is peritoneum. The question is really only asking which structure lies posterior to the rectus abdominis muscle. Since this region Is below the arcuate line, the transversalis fascia and peritoneum lie posterior to it. Bucks fascia lies in the penis, | @ | Improve muscle * Rectus abdomi The rectus sheath is formed by the aponeuroses of the lateral abdominal wall muscles. The rectus sheath has a composition that varies according to anatomical level, 1. Above the costal margin the anterior sheath is composed of external oblique aponeurosis, the costal cartilages are posterior to it. 2. From the costal margin to the arcuate line, the anterior rectus sheath is composed of external oblique aponeurosis and the anterior part of the internal oblique aponeurosis. The posterior part of the internal oblique aponeurosis and transversus abdominis form the posterior rectus sheath, 3, Below the arcuate line the aponeuroses of all the abdominal muscles lie in anterior aspect of the rectus sheath. Posteriorly lies the transversalis fascia and peritoneum. The arcuate line Is the point at which the inferior epigastric vessels enter the rectus sheath. Soue mu nate ° Question 414 of 611 x p eo In a patient with a carcinoma of the distal sigmoid colon, what Is the most likely source of its blood supply? Hleocolic artery External iliac artery [Bens iliac artery Superior mesenteric artery During a high anterior resection of such tumours, the inferior mesenteric artery is ligated. Note that the branches (mainly middle rectal branch) of the internal iliac artery are important in maintaining vascularity of the rectal stump and hence the integrity of the anastomoses, Improve Rectum * The rectum Is approximately 12 cm long. It is a capacitance organ. It has both intra and extraperitoneal components. The transition between the sigmoid colon is marked by the disappearance of the tenia coli.The extra peritoneal rectum is surrounded by mesorectal fat that also contains lymph nodes. This mesorectal fatty layer is removed surgically during rectal cancer surgery (Total Mesorectal Excision). The fascial layers that surround the rectum are important clinical landmarks, anteriorly ies the fascia of Denonvilliers. Posteriorly ies Waldeyers fascia, Extra peritoneal rectum *+ Posterior upper third * Posterior and lateral middle third + Whole lower third ‘Anteriorly (Males) Rectovesical pouch Bladder Prostate Seminal vesicles Anteriorly (Females) Recto-uterine pouch (Douglas) Cervix Vaginal wall Posteriorly Sacrum Coccyx Middle sacral artery Laterally Levator ani Coccygeus Arterial supply Superior rectal artery Venous drainage Superior rectal vein Lymphatic drainage + Mesorectal lymph nodes (superior to dentate line) * Inguinal nodes (Inferior to dentate line) [save notes Search Search textbook Q Google search on "Rectum" + uggest lnk Suggest media Dashboard ReRK ORE ert 3° Question 415 of 611 v p °o What is the largest branch of the brachial artery? Radial artery Ulnar artery Humeral nutrient artery Uinar collateral artery The profunda brachii artery is the largest branch and then continues in the radial groove of the humerus, | | Improve Brachial artery * The brachial artery begins at the lower border of teres major asa continuation of the axillary artery. It terminates in the cubital fossa at the level of the neck of the radius by dividing Into the radial and ulnar arteries. Relations Posterior relations include the long head of triceps with the radial nerve and profunda vessels intervening. Anteriorly it is overlapped by the medial border of biceps. Itis crossed by the median nerve in the middle of the arm. In the cubital fossa itis separated from the median cubital vein by the bicipital aponeurosis. The basilic vein is in contact at the most proximal aspect of the cubital fossa and lies medially. Ne = Tr & @ © Save my notes Search ‘Search textbook B EE eo Question 416 of 611 x Pp © A 23 year old man falls over whilst intoxicated and a shard of glass transects his median nerve at the proximal border of the flexor retinaculum. His tendons escape injury. Which of the following features is least likely to be present? Loss of power of opponens pollicis Parasthesia of the palmar aspect of the second digit Loss of power of abductor pollicis brevis The median nerve may be injured proximal to the flexor retinaculum. This will result in loss of abductor pollicis brevis, flexor pollicis brevis, opponens pollicis and the first and second lumbricals. When the patient is asked to close the hand slowly there is a lag of the index and middle fingers reflecting the impaired lumbrical muscle function. The sensory changes are minor and do not extend to the dorsal aspect of the thenar eminence. Abductor pollicis longus will contribute to thumb abduction (and is innervated by the posterior Interosseous nerve) and therefore abduction will be weaker than prior to the injury. we | @ | improve Ne n> Median nerve * The median nerve is formed by the union of a lateral and medial root respectively from the lateral (C56,7) and medial (C8 and T1) cords of the brachial plexus; the medial root passes anterior to the third part of the axillary artery. The nerve descends lateral to the brachial artery, crosses to its medial side (usually passing anterior to the artery). It passes deep to the bicipltal aponeurosis and the median cubital vein at the elbow. It passes between the two heads of the pronator teres muscle, and runs on the deep surface of flexor digitorum superficialis (within its fascial sheath). Near the wrist it becomes superficial between the tendons of flexor digitorum superficialis and flexor carpi radialis, deep to palmaris longus tendon. It passes deep to the flexor retinaculum to enter the palm, but lies anterior to the long flexor tendons within the carpal tunnel, Branches Region Branch Upper No branches, although the nerve commonly communicates with the arm musculocutaneous nerve Forearm — Pronator teres Pronator quadratus Flexor carpi radialis, Palmaris longus Flexor digitorum superficialis Flexor pollicis longus Flexor digitorum profundus (only the radial half) Distal Palmar cutaneous branch forearm Hand Motor supply (LOAF) (Motor) * Lateral 2 lumbricals * Opponens pollicis * Abductor pollicis brevis * Flexor pollicis brevis, Hand * Over thumb and lateral 2 % fingers (Sensory) =» _On the palmar aspect this projects proximally, on the dorsal aspect only the distal regions are innervated with the radial nerve providing the more proximal cutaneous innervation Patterns of damage Damage at wrist + eg. carpal tunnel syndrome * paralysis and wasting of thenar eminence muscles and opponens pollicis (ape hand deformity) + sensory loss to palmar aspect of lateral (radial) 2 % fingers. Damage at elbow, as above plus: + unable to pronate forearm + weak wrist flexion + ulnar deviation of wrist Anterior interosseous nerve (branch of median nerve) + leaves just below the elbow + results in loss of pronation of forearm and weakness of long flexors of thumb and index finger Topography of the median nerve Lateral pectoral nerve Topography of the median nerve Lateral pectoral nerve __ Medial pectoral nerve Musculocutaneous nerve Radial nerve Deep branch of radial Superficial branch of ‘radial Median > Tr By & o Save my notes °o Question 417 of 611 v 8 So 45 year old man is stabbed in the abdomen and the inferior vena cava Is injured. How many functional valves does this vessel usually have? Eo 1 eo 3 eo 2 @ 4 oe Mnemonic for the Inferior vena cava tributaries: | Like To Rise So High: Maes Lumbar Testicular Renal Suprarenal Hepatic vein ‘The lack of valves in the IVC Is important clinically when It is cannulated during cardiopulmonary bypass, using separate SVC and IVC catheters, such as when the right atrium is to be opened Note that there Is a non functional valve between the right atrium and inferior vena cava. Inferior vena cava * Origin +5 Path * Left and right common iliac veins merge to form the IVC. + Passes right of midline * Paired segmental lumbar veins drain into the IVC throughout its length * The right gonadal vein empties directly into the cava and the left gonadal vein generally empties into the left renal vein. + The next major veins are the renal veins and the hepatic veins * Pierces the central tendon of diaphragm at T8 + Right atrium 2p Relations Anteriorly Small bowel, fist and third part of duodenum, head of pancreas, liver and bile duct, right common iliac artery, right gonadal artery Posteriorly Right renal artery, right psoas, right sympathetic chain, coeliac ganglion Levels Level ve 18 Hepatic vein, inferior phrenic vein, pierces diaphragm u Right suprarenal vein, renal vein 2 Gonadal vein us Lumbar veins Ls ‘Common iliac vein, formation of IVC Bie @-+ =e = Tr & wo Save my notes Search ‘Search textbook ° Question 418 of 611 v B } Which muscle is supplied by the superficial peroneal nerve? Peroneus tertius Sartorius ‘Adductor magnus oe Gracilis e at | | improve | Superficial peroneal nerve * Supplies + Lateral compartment of leg: peroneus longus, peroneus brevis (action: eversion and plantar flexion) + Sensation over dorsum of the foot (except the first web space, which is innervated by the deep peroneal nerve) Path + Passes between peroneus longus and peroneus brevis along the length of the proximal one third of the fibula + 10-12 cm above the tip of the lateral malleolus, the superficial peroneal nerve pierces the fascia + 6-7 cm distal to the fibula, the superficial peroneal nerve bifurcates into intermediate and medial dorsal cutaneous nerves Biew- ‘Save my notes Search ° Question 419 of 611 ¥ n ° Which of the muscles below does not cause lateral rotation of the hip? Obturator internus Quadratus femoris Gemellus inferior Piriformis: Es - Mnemonic lateral hip rotators: P-GO-GO-Q (top to bottom) Piriformis Gemellus superior Obturator internus: Gemellus inferior Obturator externus. Quadratus femoris Pectineus adducts and medially rotates the femur. | | improve Hip joint * + Head of femur articulates with acetabulum of the pelvis + Both covered by articular hyaline cartilage * The acetabulum forms at the union of the illum, pubis, and ischium + The triradiate cartilage (¥-shaped growth plate) separates the pelvic bones + The acetabulum holds the femoral head by the acetabular labrum + Normal angle between femoral head and femoral shaft is 130° Ligaments + Transverse ligament: joints anterior and posterior ends of the articular cartilage + Head of femur ligament (ligamentum teres): acetabular notch to the fovea. Contains arterial supply to head of femur in children. A Extracapsular ligaments * lliofemoral ligament: inverted Y shape. Anterior iliac spine to the trochanteric line + Pubofemoral ligament: acetabulum to lesser trochanter * Ischiofemoral ligament: posterior support. Ischium to greater trochanter. Blood supply Medial circumflex femoral and lateral circumflex femoral arteries (Branches of profunda femoris). Also from the inferior gluteal artery. These form an anastomosis and travel to up the femoral neck to supply the head, [seve yi] Qo Question 420 of 611 v PP oO 21 year old man is stabbed in the antecubital fossa. A decision is made to surgically explore the wound. At operation, the surgeon dissects down onto the brachial artery. A nerve is identified medially, which nerve is it likely to be? Radial e Recurrent branch of median Anterior interosseous Ulnar "@ | Improve Median nerve * ‘The median nerve is formed by the union of a lateral and medial root respectively from the lateral (C5,6,7) and medial (C8 and T1) cords of the brachial plexus; the medial root passes anterior to the third part of the axillary artery. The nerve descends lateral to the brachial artery, crosses to its, ‘medial side (usually passing anterior to the artery). It passes deep to the bicipital aponeurosis and the median cubital vein at the elbow. It passes between the two heads of the pronator teres muscle, and runs on the deep surface of flexor digitorum superficialis (within its fascial sheath). ‘Near the wrist it becomes superficial between the tendons of flexor digitorum superficialls and flexor carpi radialis, deep to palmaris longus tendon. it passes deep to the flexor retinaculum to enter the palm, but lies anterior to the long flexor tendons within the carpal tunnel. Region Branch Upper No branches, although the nerve commonly communicates with the arm musculocutaneous nerve Forearm — Pronator teres Pronator quadratus Flexor carpi radialis Palmaris longus Flexor digitorum superficialis Flexor pollicis longus Flexor digitorum profundus (only the radial half) Distal Palmar cutaneous branch forearm Hand Motor supply (LOAF) (Motor) * Lateral 2 lumbricals + Opponens pollicis * Abductor pollicis brevis * Flexor pollicis brevis. Hand + Over thumb and lateral 2 % fingers (Sensory) + On the palmar aspect this projects proximally, on the dorsal aspect only the distal regions are innervated with the radial nerve providing the more proximal cutaneous innervation, Patterns of damage Damage at wrist + e.g. carpal tunnel syndrome * paralysis and wasting of thenar eminence muscles and opponens pollicis (ape hand deformity) + sensory loss to palmar aspect of lateral (radial) 2 % fingers Damage at elbow, as above plus: * unable to pronate forearm = weak wrist flexion + ulnar deviation of wrist Anterior interosseous nerve (branch of median nerve) + leaves just below the elbow + results in loss of pronation of forearm and weakness of long flexors of thumb and index finger Topography of the median nerve Lateral pectoral nerve Medial pectoral nerve ~ Musculocutaneous nerve Topography of the median nerve Lateral pectoral nerve Medial pectoral nerve Musculocutaneous nerve Median nerve Radial nerve Deep branch of radial Superficial branch of radial tinar Median Bie f&- 7 Tr By @ Save my notes eo Question 421 of 611 v , S Which of the following statements relating to the posterior cerebral artery is false? It supplies the visual cortex It is closely related to the 3rd cranial nerve When occluded may result in contralateral loss of field of vision e oe Itis a branch of the basilar artery @ @Q eo The posterior cerebral arteries are formed by the bifurcation of the basilar artery and is. connected to the circle of Willis via the posterior communicating artery. The posterior cerebral arteries supply the occipital lobe and part of the temporal lobe [ot | | imorove | Circle of Willis * The two internal carotid arteries and two vertebral arteries form an anastomosis known as the Circle of Willis on the inferior surface of the brain. Each half of the circle is formed by: 1. Anterior communicating artery 2. Anterior cerebral artery 3. Internal carotid artery 4, Posterior communicating artery 5. Posterior cerebral arteries and the termination of the basilar artery The circle and its branches supply; the corpus striatum, internal capsule, diencephalon and midbrain, Vertebral arteries * Enter the cranial cavity via foramen magnum * Lie in the subarachnoid space + Ascend on anterior surface of medulla oblongata * Unite to form the basilar artery at the base of the pons Branches: + Posterior spinal artery * Anterior spinal artery + Posterior inferior cerebellar artery Basilar artery Branches: * Anterior inferior cerebellar artery + Labyrinthine artery + Pontine arteries * Superior cerebellar artery + Posterior cerebral artery Internal carotid arteries Branches: + Posterior communicating artery * Anterior cerebral artery + Middle cerebral artery + Anterior choroid artery eo Question 422 of 611 ¥ Bp So Abuilder falls off a ladder whilst laying roof tiles. He sustains a burst fracture of L2. The MRI ‘scan shows complete nerve transection at this level, as a result of the injury. Which clinical sign will not be present initially? Flaceid paralysis of the legs Sensory loss in the legs Incontinence Areflexia In lower motor neuron lesions everything is reduced The main purpose of this question is to differentiate the features of an UMN lesion and a LMN lesion. The features of a LMN lesion include: ‘+ Flaccid paralysis of muscles supplied ‘+ Atrophy of muscles supplied. + Loss of reflexes of muscles supplied ‘+ Muscles fasciculation For lesions below L1 LMN signs will occur. Hence in an L3 lesion, there will be loss of the patella reflex but there will be no extensor plantar reflex. @ | F | Improve Spinal cord « + Located in a canal within the vertebral column that affords it structural support. ‘+ Rostrally it continues to the medulla oblongata of the brain and caudally it tapers at a level corresponding to the L1-2 interspace (in the adult), a central structure, the filum terminale anchors the cord to the first coccygeal vertebra ‘+ The spinal cord is characterised by cervico-lumbar enlargements and these, broadly speaking, are the sites which correspond to the brachial and lumbar plexuses respectively. There are some key points to note when considering the surgical anatomy of the spinal cord: * During foetal growth the spinal cord becomes shorter than the spinal canal, hence the adult site of cord termination at the L1-2 level * Due to growth of the vertebral column the spine segmental levels may not always correspond to bony landmarks as they do in the cervical spine. * The spinal cord is incompletely divided into two symmetrical halves by a dorsal median sulcus and ventral median fissure. Grey matter surrounds a central canal that is continuous rostrally with the ventricular system of the CNS. * The grey matter is sub divided cytoarchitecturally into Rexeds laminae. * afferent fibres entering through the dorsal roots usually terminate near their point of entry but may travel for varying distances in Lissauers tract. In this way they may establish synaptic connections over several levels * At the tip of the dorsal horn are afferents associated with nociceptive stimul, The ventral horn contains neurones that innervate skeletal muscle The key point to remember when revising CNS anatomy is to keep a clinical perspective in mind, So itis worth classifying the ways in which the spinal cord may become injured. These include: + Trauma either direct or as a result of disc protrusion + Neoplasia either by direct invasion (rare) or as a result of pathological vertebral fracture + Inflammatory diseases such as Rheumatoid disease, or OA (formation of osteophytes compressing nerve roots ete. * Vascular either as a result of stroke (rare in cord) or as complication of aortic dissection * Infection historically diseases such as TB, epidural abscesses. The anatomy of the cord will, o an extent dictate the clinical presentation. Some points/ conditions to remember: + Brown- Sequard syndrome-Hemisection of the cord producing ipsilateral loss of proprioception and upper motor neurone signs, plus contralateral loss of pain and. temperature sensation. The explanation of this is that the fibres decussate at different levels. * Lesions below L1 will tend to present with lower motor neurone signs Tr gy @ © Search Seach aboot a 8 Question 423 of 611 v Bp 1 ‘A 23 year old man is involved in a fight outside a nightclub and sustains a laceration to his right arm. On examination, he has lost extension of the fingers in his right hand, Which of the nerves listed below is most likely to have been divided? Median Musculocutaneous Uinar Axillary ‘The radial nerve supplies the extensor muscle group. '& | improve | Radial nerve * Continuation of posterior cord of the brachial plexus (root values C5 to T1) Path + Inthe axilla: lies posterior to the axillary artery on subscapularis, latissimus dorsi and teres major. * Enters the arm between the brachial artery and the long head of triceps (medial to humerus). * Spirals around the posterior surface of the humerus in the groove for the radial nerve. * At the distal third of the lateral border of the humerus it then plerces the intermuscular septum and descends in front of the lateral epicondyle. + Atthe lateral epicondyle it lies deeply between brachialis and brachioradialis where it then divides into a superficial and deep terminal branch. *+ Deep branch crosses the supinator to become the posterior interosseous nerve. In the image below the relationships of the radial nerve can be appreciated In the image below the relationships of the radial nerve can be appreciated Regions innervated Motor (main nerve) Motor (posterior Interosseous branch) Sensory + Teeps + Anconeus + Brachioradiais ‘Exensor carp radial ‘© supinator ‘+ dtensor carp ulnaris Extensor digitorum Extensor indicts Extensor digit minim Extensor pollicis longus and brevis ‘Abductor pollicis longus ‘The atea of skin supplying the proximal phalanges on the dorsel aspect of the hand is supplied by the ‘agial nerve (this does not apply to the litle finger and part ofthe rng finge!) Muscular innervation and effect of denervation Anatomical location Shoulder Arm Forearm Muscle affected Long head of triceps Triceps Supinator Brachioradialis Extensor carpi radialis longus and brevis Effect of paralysis Minor effects on shoulder stability In abduction Loss of elbow extension Weakening of supination of prone hand and elbow fiexion in mid prone position The cutaneous sensation of the upper limb- illustrating the contribution of the radial nerve The cutaneous sensation of the upper limb- illustrating the contribution of the radial nerve wero tee = a wear eto a = anon rostinoe Bie @®- #22 Tr & @ Save my notes Search ‘Search textbook Q Google search on "Radial nerve" + Suggest nk Suggest media Dashboard aO8 °e Question 424 of 611 x Bp oO ‘A.34 year old man with a submandibular gland stone Is undergoing excision of the submandibular gland. The incision is sited transversely approximately 4cm below the mandible. After incising the skin, platysma and deep fascia which of the following structures is most likely to be encountered. Facial artery a @ a Hypoglossal nerve Glossopharyngeal nerve e When approaching the submandibular gland the facial vein and submandibular iymph nodes are the most superficially encountered structures. Each sub mandibular gland has a superficial and deep part, separated by the mylohyoid muscle. The facial artery passes in a groove on the superficial aspect of the gland. It then emerges onto the surface of the face by passing between the gland and the mandible. The facial vein is encountered first in this surgical approach because the incision is made 4cm below the mandible (to avoid injury to the marginal mandibular nerve) 99 | improve Submandibular gland * Relations of the submandibular gland ‘Superfici Platysma, deep fascia and mandible ‘Submandibular lymph nodes Facial vein (facial artery near mandible) Marginal mandibular nerve Cervical branch of the facial nerve Deep Facial artery (inferior to the mandible) Mylohyoid muscle Sub mandibular duct Hyoglossus muscle Lingual nerve ‘Submandibular ganglion Hypoglossal nerve Submandibular duct (Wharton's duct) *+ Opens lateral to the lingual frenulum on the anterior floor of mouth. * Semlength + Lingual nerve wraps around Wharton's duct. As the duct passes forwards it crosses medial to the nerve to lie above it and then crosses back, lateral to it, to reach a position below the nerve. Innervation ‘+ Sympathetic Innervation- Derived from superior cervical ganglion + Parasympathetic innervation- Submandibular ganglion via lingual nerve Arterial supply Branch of the facial artery. The facial artery passes through the gland to groove its deep surface. Itthen emerges onto the face by passing between the gland and the mandible. Venous drainage Anterior facial vein (ies deep to the Marginal Mandibular nerve) Lymphatic drainage Deep cervical and jugular chains of nodes Bie um- Tr By @ © Search Search textbook Q Google search on “Submandibular gland” Suggest link Suggest media Dashboard aRona eeene eo Question 425 of 611 v Bp ° A 18 year old man presents with an indirect inguinal hernia and undergoes surgery. The deep Inguinal ring is exposed and held with a retractor at its medial aspect. Which structure is most likely to lie under the retractor? Ureter Internal iliac vein Femoral artery Lateral border of rectus abdominis a Boundarles of the deep inguinal ring: * Superolaterally - transversalis fascia ‘+ Inferomedially - inferior epigastric artery The deep inguinal ring is closely related to the inferior epigastric artery. The inferior epigastric artery forms part of the structure referred to as Hesselbach's triangle. @ | @ | Improve Inguinal canal * Inguinal canal * Location * Above the inguinal ligament + The inguinal canal Is 4m long + The superficial ring is located anterior to the pubic tubercle + The deep ring is located approximately 1.5:2cm above the half way point between the anterior superior iliac spine and the pubic tubercle Boundaries of the inguinal canal Floor + External oblique aponeurosis, + Inguinal ligament + Lacunar ligament Roof + Internal oblique + Transversus abdominis External oblique aponeurosis, + Transversalls fascia + Conjoint tendon Laterally + Internal ring + Transversalis fascia + Fibres of internal oblique Medially + External ring + Conjoint tendon Contents Males Spermatic cord and ilioinguinal As it passes through the canal the spermatic nerve cord has 3 coverings: + External spermatic fascia from external oblique aponeurosis + Cremasteric fascia + Internal spermatic fascia Females Round ligament of uterus and ilioinguinal nerve Related anatomy of the inguinal region ‘The boundaries of Hesselbachs triangle are commonly tested and illustrated below: The image below demonstrates the close relationship of the vessels to the lower limb with the inguinal canal. A fact to be borne in mind when repairing hernial defects in this region. Bilew&- Tr & @ @ ‘Search sear ebook gg Q Google search on "Inguinal canal” (<) Question 426 of 611 ¥ n Oo ‘76 year old man is undergoing an abdominal aortic aneurysm repair. The surgeons occlude the aorta with two clamps, the inferior clamp being placed at the point of aortic bifurcation. Which of the following vertebral bodies will ie posterior to the clamp at this level? u T10 Ls 2 The aorta bifurcates at L4. An important landmark that is tested frequently, Abdominal aorta * Abdominal aortic topography Origin Termination Posterior relations Anterior relations Right lateral relations Left lateral relations m2 4 L1-L4 Vertebral bodies Lesser omentum Liver Left renal vein Inferior mesenteric vein Third part of duodenum Pancreas Parietal peritoneum Peritoneal cavity Right crus of the diaphragm Cisterna chyli IVC (becomes posterior distally) 4th part of duodenum Duodenal-jejunal flexure Left sympathetic trunk The abdominal aorta BTS ae tT & Be Save my notes Search Search textbook Q Google search on “Abdominal aorta® Suggest ink “Suggest media Dashboard Sena ORS e Question 427 of 611 . : ° 34 year old lady undergoes a thyroidectomy for Graves disease, Post operatively she develops a tense haematoma in the neck. In which of the following fascial planes will it be contained? Gerotas fascia Waldeyers fascia Sibsons fascia Clavipectoral fascia The pretracheal fascia encloses the thyroid and is unyielding. Therefore tense haematomas can develop. | | imorove | Thyroid gland * + Right and left lobes connected by isthmus + Surrounded by sheath from pretracheal layer of deep fascia + Apex: Lamina of thyroid cartilage ‘+ Base: 4th-Sth tracheal ring + Pyramidal lobe: from isthmus + May be attached to foramen caecum at the base of the tongue Relations Anteromedially * Stemothyroid ‘Superior belly of omohyoid * Stemohyoid Anterior aspect of sternocleidomastoid Posterolaterally Carotid sheath Medially + Larynx * Trachea + Pharynx + esophagus * Cricothyroid muscle + External laryngeal nerve (near superior thyroid artery) + Recurrent laryngeal nerve (near inferior thyroid artery) Posterior + Parathyroid glands ‘+ Anastomosis of superior and inferior thyroid arteries Isthmus + Anteriorly: Sternothyroids, sternohyolds, anterior jugular veins + Posteriorly: 2nd, 3rd, th tracheal rings (attached via Ligament of, Berry) Blood Supply Arterial * Superior thyroid artery (1st branch of external carotid) * Inferior thyroid artery (from thyrocervical trunk) + Thyroidea ima (in 10% of population -from brachiocephalic artery or aorta) Venous + Superior and middle thyroid veins - into the NV * Inferior thyroid vein - into the brachiocephalic veins Teceaes ii MW 7 Tr & @ @ Search ‘Search textbook Q Google search on “Thyroid gland” Suggest tink suggest media Dashboard ea O6 (<) Question 428 of 611 v Bp oO Which of the following muscles is not innervated by the deep branch of the ulnar nerve? Adductor pollicis Hypothenar muscles All the interosseous muscles Third and fourth lumbricals Opponens pollicis Is Innervated by the deep branch of the median. If you answered the question incorrectly, note that it asks about which muscle is NOT innervated by the deep branch @ | | Improve Ulnar nerve « + caTt Supplies (no muscles in the upper arm) * Flexor carpi ulnaris, + Flexor digitorum profundus * Flexor digiti minimi * Abductor digiti minim! + Opponens digit! minim! + Adductor pollicis + Interossei muscle + Third and fourth lumbricals Palmaris brevis Path + Posteromedial aspect of upper arm to flexor compartment of forearm, then along the ulnar. Passes beneath the flexor carpi ulnaris muscle, then superficially over the flexor retinaculum into the palm of the hand, Unarnerve x. Branches Branch ‘Suppl Muscular branch Flexor carpi ulnaris Medial half of the flexor digitorum profundus Palmar cutaneous branch (Arises near _Skin on the medial part of the palm the middle of the forearm) Dorsal cutaneous branch Dorsal surface of the medial part of the hand Superficial branch Cutaneous fibres to the anterior surfaces of the medial one and one-half digits Deep branch Hypothenar muscles All the interosseous muscles Third and fourth lumbricals, Adductor pollicis Medial head of the flexor pollicis brevis Effects of injury Damage at the wrist ‘+ Wasting and paralysis of intrinsic hand muscles (claw hand) ‘+ Wasting and paralysis of hypothenar muscles ‘+ Loss of sensation medial 1 and half fingers. Damage at the elbow ‘+ Radial deviation of the wrist ‘+ Clawing less in 4th and Sth digits 3° Question 429 of 611 v p } 'A.35 year tennis player attends clinic reporting tingling down his arm. He says that his funny bone’ was hit very hard by a tennis ball. There is weakness of abduction and adduction of his extended fingers. Which nerve has been affected? Anterior interosseous Posterior interosseous Median Musculocutaneous ‘The ulnar nerve arises from the medial cord of the brachial plexus (C8, T1 and contribution from C7). The nerve descends between the axillary artery and vein, posterior to the cutaneous nerve of the forearm and then lies anterior to triceps on the medial side of the brachial artery. In the distal half of the arm it passes through the medial intermuscular septum, and continues between this structure and the medial head of triceps to enter the forearm between the medial epicondyle of the humerus and the olecranon. It may be injured at this site in this scenario. “9 Improve } Ulnar nerve * origin + cati Supplies (no muscles in the upper arm) * Flexor carpi ulnaris * Flexor digitorum profundus * Flexor digiti minimi * Abductor digit! minimi + Opponens digit! minimi * Adductor pollicis + Interossel muscle * Third and fourth tumbricals + Palmaris brevis Path + Posteromedial aspect of upper arm to flexor compartment of forearm, then along the ulnar. Passes beneath the flexor carpi ulnaris muscle, then superficially over the flexor retinaculum into the palm of the hand, Branches Branch Muscular branch Palmar cutaneous branch (Arises near the middle of the forearm) Dorsal cutaneous branch Superficial branch ‘Supplies Flexor carpi ulnaris Medial half of the flexor digitorum profundus Skin on the medial part of the palm Dorsal surface of the medial part of the hand Cutaneous fibres to the anterior surfaces of the medial one and one-half digits Deep branch Hypothenar muscles All the interosseous muscles Third and fourth lumbricals, ‘Adductor pollicis Medial head of the flexor pollicis brevis Effects of injury Damage at the wrist ‘+ Wasting and paralysis of intrinsic hand muscles (claw hand) ‘+ Wasting and paralysis of hypothenar muscles + Loss of sensation medial 1 and half fingers Damage at the elbow ‘+ Radial deviation of the wrist + Clawing less in 4th and Sth digits BleaW- #8 5° Tr & @ o | Save my notes Search ‘Search textbook ‘Q Google search on "Uinarnerve" Suggest lnk Suggest media Dashboard u 12 13 14 15 16 7 18 3 x Rue C6 CSCC << Coe ee ee ° aveston 430 of 611 - 5 >) Which of the following structures are not closely related to the adductor longus muscle? Long saphenous vein ‘The profunda branch of the femoral artery 16% Pectineus muscle Femoral nerve Femoral triangle: ‘Adductor longus medially Inguinal ligament superiorly Sartorius muscle laterally Adductor longus forms the medial border of the femoral triangle. It is closely related to the long saphenous vein which overlies it and the profunda branch of the femoral artery. The femoral nerve is related to tt inferiorly. However, the tendon of illacus inserts proximally and is not in contact with adductor longus. we | @ | improve Adductor longus * Origin Anterior body of pubis, Insertion Middle third of linea aspera Action Adducts and flexes the thigh, medially rotate the hip Innervation Anterior division of obturator nerve (L2,L3, L4) The schematic image below demonstrates the relationship of the adductor muscles, Prose major tiacus ‘ramen Piriformis favciae latae The schematic image below demonstrates the relationship of the adductor muscles ao mmaer cus \ fascias tae tines onl tract | Adductor magnus 4 Bieu- Tr &y @ Search Search textbook Go Q Google search on *Adductor longus* + suggest nk + suggest media Dashboard lv °o Question 431 of 611 ¥ 5 © 65 year old man with long standing atrial fibrillation develops an embolus to the lower leg. The decision is made to perform an embolectomy, utilising a trans popliteal approach. After incising the deep fascia, which of the following structures will the surgeons encounter first on exploring the central region of the popliteal fossa? Popliteal vein ‘Common peroneal nerve Popliteal artery None of the above 2 The tibial nerve lies superior to the vessels in the inferior aspect of the popliteal fossa. in the Upper part of the fossa the tibial nerve lies lateral to the vessels, it then passes superficial to them to lie medially. The popliteal artery is the deepest structure in the popliteal fossa [|e | improve } Popliteal fossa * Boundaries of the popliteal fossa Laterally Biceps femoris above, lateral head of gastrocnemius and plantaris below Medially Semimembranosus and semitendinosus above, medial head of gastrocnemius below Floor Popliteal surface of the femur, posterior ligament of knee joint and popliteus muscle Roof ‘Superficial and deep fascia Image showing the popliteal fossa Contents + Popliteal artery and vein + Small saphenous vein + Common peroneal nerve + Tibial nerve * Posterior cutaneous nerve of the thigh + Genicular branch of the obturator nerve + Lymph nodes Bia w&- it Search ‘Search textbook Q Google search on *Popliteal fossa” suggest tink Suggest media Dashboard n 12 13 14 GAS CEC ee 40 rae: °o Question 432 of 611 v 5 © Which of the following is true in connection with the phrenic nerves? ‘They are derived from spinal roots C 2,3,4 They pierce the diaphragm at the level of T7 They consist of motor fibres only None of the above 634.5, Keeps the diaphragm alive! They both lie anterior to the hilum of the lung. The phrenic nerves have both motor and sensory functions. For this reason sub diaphragmatic pathology may cause referred pain to the shoulder. (2 | [oe] Phrenic nerve * origin + 0345 ‘Supplies ‘= Diaphragm, sensation central diaphragm and pericardium Path ‘+ The phrenic nerve passes with the internal jugular vein across scalenus anterior. It passes deep to prevertebral fascia of deep cervical fascia. + Left: crosses anterior to the 1st part of the subclavian artery. ‘+ Right: Anterior to scalenus anterior and crosses anterior to the 2nd part of the subclavian artery. ‘+ On both sides, the phrenic nerve runs posterior to the subclavian vein and posterior to the internal thoracic artery as it enters the thorax. Right phrenic nerve + Inthe superior mediastinum: anterior to right vagus and laterally to superior vena cava ‘+ Middle mediastinum: right of pericardium + Itpasses over the right atrium to exit the diaphragm at T8 Left phrenic nerve + Passes lateral to the left subclavian artery, aortic arch and left ventricle + Passes anterior to the root of the lung * Pierces the diaphragm alone Image showing the passage of the phrenic nerve in the neck x oa ed Search ‘Search textbook Ba Q Google search on "Phrenic nerve" suggest lnk Suggest media Dashboard 3° Question 433 of 611 ¥ p ° ‘A10 year old child has a grommet inserted for a glue ear. What type of epithelium is present on the external aspect of the tympanic membrane? Ciliated columnar Non ciliated columnar Non stratified squamous None of the above ‘The external aspect of the tympanic membrane is lined by stratified squamous epithelium. This is ‘significant clinically in the development of middle ear infections when this type of epithelium may migrate inside the middle ear. Ear- anatomy * ‘The ear is composed of three anatomically distinct regions. External ear Auricle is composed of elastic cartilage covered by skin. The lobule has no cartilage and contains fat and fibrous tissue. External auditory meatus is approximately 2.5cm long, Lateral third of the external auditory meatus is cartilaginous and the medial two thirds is bony. ‘The region is innervated by the greater auricular nerve. The auriculotemporal branch of the trigeminal nerve supplies most of the external auditory meatus and the lateral surface of the auricle. Middle ear ‘Space between the tympanic membrane and cochlea, The aditus leads to the mastoid air cells is the route through which middle ear infections may cause mastoiditis. Anteriorly the eustacian tube connects the middle ear to the naso pharynx. The tympanic membrane consists of: + Outer layer of stratified squamous epithelium, ‘+ Middle layer of fibrous tissue. ‘+ Inner layer of mucous membrane continuous with the middle ear. The tympanic membrane is approximately 1om in diameter. ‘The chorda tympani nerve passes on the medial side of the pars flaccida ‘The middle ear is innervated by the glossopharyngeal nerve and pain may radiate to the middle ear following tonsillectomy. Ossicles Malleus attaches to the tympanic membrane (the Umbo) Malleus articulates with the incus (synovial joint). Incus attaches to stapes (another synovial joint). Internal ear Cochlea, semi circular canals and vestibule Organ of corti is the sense organ of hearing and is located on the inside of the cochlear duct on the basilar membrane. Vestibule accommodates the utricule and the saccule. These structures contain endolymph and are surrounded by perilymph within the vestibule, ‘The semicircular canals lie at various angles to the petrous temporal bone. All share a common Bor se = Save my notes Search ‘Search textbook. B Q Google search on *Ear- anatomy" + Suage “suggest media Dashboard 46 ‘A 75-year-old man presents with hepatomegaly and ascites. A CT scan shows evidence of post hepatic portal hypertension. The inferior vena cava passes through the diaphragm at which vertebral level? 15 To mm eo eo 19 e QS eo Itpasses through the diaphragm at T8. wo | | Improve Inferior vena cava * ‘+ Left and right common iliac veins merge to form the IVC. + Passes right of midline + Paired segmental lumbar veins drain into the IVC throughout its length ‘+ The right gonadal vein empties directly into the cava and the left gonadal vein generally empties into the left renal vein. ‘+ The next major veins are the renal veins and the hepatic veins ‘+ Pierces the central tendon of diaphragm at T8 + Right atrium Relations Anteriorly Small bowel, first and third part of duodenum, head of pancreas, liver and bile duct, right common iliac artery, right gonadal artery Posteriorly Right renal artery, right psoas, right sympathetic chain, coellac ganglion Levels Level Vein 18 Hepatic vein, inferior phrenic vein, pierces diaphragm u Right suprarenal vein, renal vein LB Gonadal vein Ls Lumbar veins ts Common iliac vein, formation of IVC n> Bia =: Tr & @ o [seve gts] Search ‘Search textbook ‘Q Google search on “inferior vena cava Suggest lnk Fuggest media Dashboard o Question 435 of 611 v Pp © Which of the following statements relating to the Cavernous Sinus is false? The pituitary gland lies medially The internal carotid artery passes through it The temporal lobe of the brain is a lateral relation | remensternencotne temilanoptcneveeon ee iam The ophthalmic veins drain into the anterior aspect of the sinus The veins that drain into the sinus are important as sepsis can cause cavernous sinus thrombosis. The maxillary branch of the trigeminal and not the mandibular branches pass through the sinus Improve Cavernous sinus * ‘The cavernous sinuses are paired and are situated on the body of the sphenoid bone. It runs from the superior orbital fissure to the petrous temporal bone. Relations Medial Lateral Puitary fossa Peter (8 Temporal abe Contents Lateral wall components (from top to bottom:) Oculomotor nerve Trochlear nerve Ophthalmic nerve Maxillary nerve Contents of the sinus (from medial to tateral:) Internal carotid artery (and sympathetic plexus) ‘Abducens nerve Blood supply Ophthalmic vein, superficial cortical veins, basilar plexus of veins posteriorly. Drains into the internal jugular vein via: the superior and inferior petrosal sinuses Save my notes Search Search textbook B Q Google search on “Cavernous sinus" ‘+ Suggest lnk + suggest media Dashboard ee) kre ene <0 Ie 26 °o Question 436 of 611 v Pe oO Which of the following statements relating to the knee joint is false? Itis the largest synovial joint in the body ‘When the knee is fully extended all ligaments (bar the anterolateral aspect of the posterior cruciate ligament) of the knee joint are taut, Rupture of the anterior cruciate ligament may result in haemarthrosis The joint is innervated by the femoral, sciatic and obturator nerves o ‘The posterior aspect is intrasynovial and the knee itself comprises the largest synovial joint in the body. it may swell considerably following trauma such as ACL injury. Which may be extremely painful owing to rich innervation from femoral, sciatic and (a smaller) contribution from the obturator nerve. During full extension all ligaments are taut and the knee is locked. Improve Knee joint * The knee joint is a synovial joint, the largest and most complicated. It consists of two condylar joints between the femur and tibia and a sellar joint between the patella and the femur. The tibiofemoral articular surfaces are incongruent, however, this is improved by the presence of the menisci. The degree of congruence is related to the anatomical position of the knee joint and is greatest in full extension Knee joint compartments ‘+ Comprised ofthe pateia/femur joint lateral and medial compartments (between femur condyles and tibia) eter ‘+ Synovial membrane and cruciate ligaments partially separate the medial and lateral compartments ++ Ugamentur patelae Patellofemoral + Actions: provides joint stabil in full extension Fibrous capsule ‘The capsule of the knee joint is a complex, composite structure with contributions from adjacent tendons. Anterior fibres proximal tothe patella. It blends with the tendinous expansions of vastus medialis, Posterior These fibres: fibres the tibial condyle fetical and run from the posterior surface of the femora condyles to the posterior agpect of Attach othe femoral and tibial condyles beyond thelr articular margins, blending with the tibial collateral Medial Sores goment Lateral fibres: Bursae Medialy ‘Attach tothe femur superior to popliteus, pass over its tendon to head of fibula and bial condyle ‘+ Subcutaneous prepatelar bursa; between patella and skin ‘+ Deep infrapatellar bursa; between tibia and patellar garment ‘Subcutaneous infrapateliar bursa; between distal bal tuberosity and skin ‘Bursa between lateral head of gastrocnemius andjoint capsule ‘+ Butsa between fibular collateral ligament and tendon of biceps femoris ‘Butea between fibular collateral ligament and tendon of popteus ‘+ Bursa between medial head of gastrocnemius and the fibrous capsule {Bursa between tibial collateral ligament and tendons of sartorius, grails and semitendinosts ‘Bursa between the tendon of semimembranosus and medial tibial condyle and medial esd of ‘gsstrocnemius Posterior Highly variable and inconsistent Ligaments Medial collateral Medial epicondyle femur to medial tibial condyle: valgus stability ligament Lateral collateral Lateral epicondyle femur to fibula head: varus stability ligament Anterior cruciate Anterior tibia to lateral intercondylar notch femur: prevents tibia ligament sliding anteriorly Posterior cruciate Posterior tibia to medial intercondylar notch femur: prevents tibia ligament sliding posteriorly Patellar ligament Central band of the tendon of quadriceps femoris, extends from patella to tibial tuberosity Quadriceps tendon Posterior cruciate ligament | Anterior cruciate ligament Medial collateral ligament Tibia 4 Menisci Medial and lateral menisci compensate for the incongruence of the femoral and tibial condyles. Composed of fibrous tissue. Medial meniscus is attached to the tibial collateral ligament. Lateral meniscus is attached to the loose fibres at the lateral edge of the joint and is separate from the fibular collateral ligament. The lateral meniscus is crossed by the popliteus tendon. Nerve supply The knee joint is supplied by the femoral, tibial and common peroneal divisions of the sciatic and by a branch from the obturator nerve. Hip pathology pain may be referred to the knee, Blood supply Genicular branches of the femoral artery, popliteal and anterior tibial arteries all supply the knee joint. 3° Question 437 of 611 ’ B Oo 53 year old man is admitted to the vascular ward for a carotid endarterectomy. His CT head report confirms a left parietal lobe infarct. What type of visual field defect might be noted? Right superior quadranopia Left superior quadranopia @ @ Right homonymous hemianopia eS Lower bitemporal hemianopia ‘Superior quadranopia = temporal lobe lesion Inferior quadranopia = parietal lobe lesion Parietal lesions cause a contralateral inferior quadranopia | | improve Visual field defects * * left homonymous hemianopia means visual field defect to the left, i.e. Lesion of right optic tract + homonymous quadrantanopias: PITS (Parletal-inferior, Temporal-Superior) * incongruous defects = optic tract lesion; congruous defects = optic radiation lesion or occipital cortex Homonymous hemianopia * Incongruous defects: lesion of optic tract * Congruous defects: lesion of optic radiation or occipital cortex + Macula sparing: lesion of occipital cortex Homonymous quadrantanopias + Superior: lesion of temporal lobe * Inferior: lesion of parietal lobe + Mnemonic = PITS (Parietal-Inferior, Temporal-Superior) * Lesion of optic chiasm *+ Upper quadrant defect > lower quadrant defect = inferior chiasmal compression, commonly a pituitary tumour + Lower quadrant defect > upper quadrant defect = superior chiasmal compression, commonly a craniopharyngioma Next question > Bile®W- 2s Tr & wo | Save my notes Search Search textbook B Q Google search on “Visual field defects" “+ Suggest lnk Suggest media Dashboard un 12 13 14 15 16 7 18 19 20 BURG IK 07 CIC Cr Ce KC Re eee ° uenton 438 of 61 # P ° 83 year old man with a chronically infected right kidney is due to undergo a nephrectomy. Which of the following structures would be encountered first during a posterior approach to the hilum of the right kidney? Right renal artery Right renal vein Inferior vena cava Right testicular vein The ureter is the most posterior structure at the hilum of the right kidney and would therefore be encountered first during a posterior approach. @ | @ | Improve Renal arteries * * The right renal artery is longer than the left renal artery * The renal vein/artery/pelvis enter the kidney at the hilum Right Anterior- IVC, right renal vein, the head of the pancreas, and the descending part of the duodenum, Left _Anterior-left renal vein, the tail of the pancreas Branches + The renal arteries are direct branches off the aorta (upper border of L2- right side and L1 - left side) + In 30% there may be accessory arteries (mainly left side). Instead of entering the kidney at the hilum, they usually pierce the upper or lower part of the organ * Before reaching the hilum of the kidney, each artery divides into four or five segmental branches (renal vein anterior and ureter posterior); which then divide within the sinus into lobar arteries supplying each pyramid and cortex. *+ Each vessel gives off some small inferior suprarenal branches to the suprarenal gland, the ureter, and the surrounding cellular tissue and muscles. Qo Question 439 of 611 v Pp © During a thyroidectomy the surgeons ligate the inferior thyroid artery. From which vessel does this structure usually originate? External carotid artery Internal carotid artery Subclavian artery Vertebral artery The inferior thyroid artery originates from the thyrocervical trunk. This is a branch of the subclavian artery. | | Improve Thyroid gland * ‘+ Right and left lobes connected by isthmus. + Surrounded by sheath from pretracheal layer of deep fascia + Apex: Lamina of thyroid cartilage + Base: 4th-Sth tracheal ring ‘+ Pyramidal lobe: from isthmus ‘+ May be attached to foramen caecum at the base of the tongue Relations Anteromedially + Sternothyroid + Superior belly of omohyoid + Stemnohyoid + Anterior aspect of sternocleidomastoid Posterolaterally Carotid sheath Medi + Larynx + Trachea + Pharynx + Oesophagus * Cricothyroid muscle + External laryngeal nerve (near superior thyroid artery) ‘+ Recurrent laryngeal nerve (near inferior thyroid artery) Posterior * Parathyroid glands + Anastomosis of superior and inferior thyroid arteries Isthmus * Anteriorly: Stemothyroids, sternohyolds, anterior jugular veins * Posteriorly: 2nd, 3rd, 4th tracheal rings (attached via Ligament of Berry) Blood Supply Arterial + Superior thyrold artery (1st branch of external carotid) * Inferior thyrold artery (from thyrocervical trunk) + Thyroldea ima (in 10% of population from brachiocephalic artery or aorta) Venous + Superior and middle thyroid veins - into the JV * Inferior thyroid vein - into the brachiocephalic veins Save my notes Search Search textbook Q Google search on "Thyroid gland” “Suggest lnk + Suqgest media #448 ° question 440 of 611 z P ° Which of the following statements relating to the vertebral column is false? There are 7 cervical vertebrae The cervical and lumbar lordosis are secondary curves developing after birth due to change in shape of the intervertebral discs The lumbar vertebrae do not have a transverse process foramina @ ‘The lumbar vertebrae receive blood directly from the aorta @ | i sone RR NER pels ote Con) The spinous process is formed by 2 laminae posteriorly. « Improve Vertebral column * * There are 7 cervical, 12 thoracic, 5 lumbar, and 5 sacral vertebrae. + The spinal cord segmental levels do not necessarily correspond to the vertebral segments. For example, while the C1 cord is located at the C1 vertebra, the C8 cord is situated at the C7 vertebra. While the T1 cord is situated at the T1 vertebra, the T12 cord is situated at the T8 vertebra. The lumbar cord Is situated between T9 and T11 vertebrae. The sacral cord is situated between the T12 to L2 vertebrae Cervical vertebrae The interface between the first and second vertebra is callled the atlanto-axis junction. The C3 cord contains the phrenic nucleus. Muscle Nerve root value Deltoid 56 Biceps 56 Wrist extensors C68 Triceps C68 Wrist flexors cert Hand muscles cet1 Thoracic vertebrae The thoracic vertebral segments are defined by those that have a rib. The spinal roots form the intercostal nerves that run on the bottom side of the ribs and these nerves control the intercostal muscles and associated dermatomes. Lumbosacral vertebrae Form the remainder of the segments below the vertebrae of the thorax. The lumbosacral spinal cord, however, starts at about T9 and continues only to L2. It contains most of the segments that innervate the hip and legs, as well as the buttocks and anal regions. Cauda Equina The spinal cord ends at L1-L2 vertebral level. The tip of the spinal cord is called the conus. Below the conus, there is a spray of spinal roots that is called the cauda equina. Injuries below L2 represent injuries to spinal roots rather than the spinal cord proper. BY 48 = Tr By a | Save my notes Search ‘Search textbook GB Q Google search on “Vertebral column’ “+ Suggest lnk Suggest media Dashboard C4 4408544 ° Question 441 of 611 ¥ B © Which of the nerves listed below is responsible for providing innervation to the lower molar teeth? Greater palatine nerve Nasopalatine nerve Zygomatic nerve Mandibular nerve ‘The branches of the lower molar and premolar teeth are supplied by branches of the inferior alveolar nerve. Those of the canine and incisors by the incisive branch of the same nerve. The gingiva and supporting structures are innervated by the lingual nerve. | @ | Improve Trigeminal nerve * The trigeminal nerve is the main sensory nerve of the head. In addition to its major sensory role, it also innervates the muscles of mastication. Distribution of the trigeminal nerve Sensory + Scalp + Face * Oral cavity (and teeth) + Nose and sinuses * Dura mater Motor ‘© Muscles of mastication * Mylohyoid + Anterior belly of digastric + Tensor tympani ‘+ Tensor palati Autonomic connections (ganglia) + Ciliary + Sphenopalatine + Otic + Submandibuler Path * Originates at the pons + Sensory root forms the large, crescentic trigeminal ganglion within Meckel's cave, and contains the cell bodies of incoming sensory nerve fibres. Here the 3 branches exit. + The motor root cell bodies are in the pons and the motor fibres are distributed via the mandibular nerve, The motor root is not part of the trigeminal ganglion Branches of the trigeminal nerve Ophthalmic nerve Sensory only Maxillary nerve Sensory only Mandibular nerve Sensory and motor Sensory Ophthalmic Exits skull via the superior orbital fissure Sensation of: scalp and forehead, the upper eyelid, the conjunctiva and comea of the eye, the nose (including the tip of the nose, except alae nasi), the nasal mucosa, the frontal sinuses, and parts of the meninges (the dura and blood vessels). Maxillary Exit skull via the foramen rotundum nerve ‘Sensation: lower eyelid and cheek, the nares and upper lip, the upper teeth and gums, the nasal mucosa, the palate and roof of the pharynx, the maxillary, ethmoid and sphenoid sinuses, and parts of the meninges. Mandibular Exit skull via the foramen ovale nerve ‘Sensation: lower lip, the lower teeth and gums, the chin and jaw (except the angle of the jaw), parts of the external ear, and parts of the meninges. Motor Distributed via the mandibular nerve. The following muscles of mastication are innervated: + Masseter + Temporalis + Medial pterygoid * Lateral pterygoid Other muscles innervated include: * Tensor veli palatini + Mylohyoid * Anterior belly of digastric + Tensor tympani Overview of the trigeminal nerve He GE a wo [ seve my notes Search Q Google search on “Trigeminal nerve" + Suggest lnk + suggest meuia °o Question 442 of 611 v p oO ‘A.63 year old lady Is undergoing an axillary sentinel lymph node biopsy as part of her breast cancer treatment. Which of the structures listed below is most likely to be encountered? Subclavian artery = e Upper cord of the brachial plexus e Lower cord of the brachial plexus Axillary nerve This can be a challenging question. A particularly careless surgeon could encounter all of these. However, during a routine level 1 axillary exploration which is where the majority of sentinel nodes will be located, the nerves most commonly encountered are the intercostobrachial nerves. Axilla Boundaries of the axilla Medially Chest wall and Serratus anterior Laterally Humeral head Floor Subscapularis Lateral border of Pectoralis major Clavipectoral fascia Derived from C5-C7 and passes behind the brachial plexus to enter the nerve (of Bell) axilla It lies on the medial chest wall and supplies serratus anterior. Its location puts it at risk during axillary surgery and damage will lead to winging of the scapula. Thoracodorsal Innervate and vascularise latissimus dors nerve and thoracodorsal trunk Axillary vein Lies at the apex of the axilla, itis the continuation of the basilic vein Becomes the subclavian vein at the outer border of the first rib, Intercostobrachial Traverse the axillary lymph nodes and are often divided during axillary nerves surgery. They provide cutaneous sensation to the axillary skin, Lymph nodes The axilla is the main site of lymphatic drainage for the breast. Save my notes Search Q Google search on “Axilla" “Suggest link Suggest media Dashboard eyanrena " 12 13 4 SRECECEE CMH ° Question 443 of 611 ’ 5 © From which embryological structure is the ureter derived? Uranchus Cloaca oe Vitello-intestinal duct oe =e = None of the above ix) ‘The ureter develops from an outpouching that arises from the mesonephric duct. The mesonephric duct Is associated with the metanephric duct that develops within the metenephrogenic blastema. This forms the site of the ureteric bud which branches off the mesonephric duct. & | @ | Improve Ureter * + 25-35 em long * Muscular tube lined by transitional epithelium + Surrounded by thick muscular coat. Becomes 3 muscular layers as it crosses the bony pelvis + Retroperitoneal structure overiying transverse processes L2-L5, + Lies anterior to bifurcation of iliac vessels * Blood supply is segmental; renal artery, aortic branches, gonadal branches, common iliac and internal iliac + Lies beneath the uterine artery Bee Tr By ww © 3° Question 444 of 611 v 5 °° Which of the structures listed below are not located within the mediastinum? Thymus eo Heart e Great vessels Arch of azygos vein o a = The vertebral bodies lie outside of the mediastinum, as do the lungs. Improve i Mediastinum * Region between the pulmonary cavities. Itis covered by the mediastinal pleura. It does not contain the lungs. It extends from the thoracic inlet superiorly to the diaphragm inferiorly Mediastinal regions + Superior mediastinum (between manubriostemal angle and T4/5) + Middle mediastinum + Posterior mediastinum + Anterior mediastinum, Region Contents ‘Superior mediastinum ‘+ Superior vena cava + Brachiocephalic veins Arch of aorta * Thoracie duct * Trachea + Oesophagus = Thymus * Vagus nerve ‘+ Left recurrent laryngeal nerve = Phrenic nerve Anterior mediastinum * Thymic remnants * Lymph nodes: * Fat * Pericardium + Heart * Aortic root + Arch of azygos vein + Main bronchi * Oesophagus + Thoracic aorta + Azygos vein + Thoracle duct * Vagus nerve + Sympathetic nerve trunks * Splanchnic nerves ‘Save my notes Search ‘Search textbook. Q Google search on “Mediastinum” Suggest ink Suggest media red Dashboard 3 Question 445 of 611 ¥ p oO ‘A35 year old man is admitted to hospital with vomiting, nausea and severe headaches. An MRI scan reveals a tumour of the cerebellopontine angle. Which one of the following pairs of cranial herves is most likely to be compressed by this tumour? Accessory and vagus e Facial and vagus e aaa 0 Glossopharyngeal and vestibulocochlear eo Vagus and vestibulocochlear ‘The cerebellopontine angle is located between the superior and inferior limbs of the angular cerebellopontine fissure formed by the petrosal cerebellar surface folding around the pons and middle cerebellar peduncle. The cerebellopontine fissure opens medially and has superior and inferior limbs that meet at a lateral apex. The fourth through the eleventh cranial nerves are located near or within the angular space between the two limbs commonly referred to as the cerebellopontine angle. The commonest lesion to affect this site is an acoustic neuroma ‘Therefore the vestibulocochlear nerve is commonly compromised. Larger lesions may also affect the facial nerve which lies closest to this site, [6 | | improve Cranial nerves * Cranial nerve lesions Olfactory nerve May be injured in basal skull fractures or involved in frontal lobe tumour extension. Loss of olfactory nerve function in relation to major CNS pathology is seldom an isolated event and thus it is poor localiser of CNS pathology. Optic nerve Problems with visual acuity may result from intra ocular disorders. Problems with the blood supply such as amaurosis fugax may produce temporary visual distortion. More important surgically Is the pupillary response to light. The pupillary size may be altered in a number of disorders. Nerves involved in the resizing of the pupil connect to the pretectal nucleus of the high midbrain, bypassing the lateral geniculate nucleus and the primary visual cortex. From the pretectal nucleus neurones pass to the Edinger - Westphal nucleus, motor axons from here pass along with the oculomotor nerve. They synapse with ciliary ganglion neurones; the parasympathetic axons from this then innervate ‘Oculomotor nerve Trochlear nerve Trigeminal nerve Abducens nerve nerve the iris and produce miosis. The miotic pupil is seen in disorders such as Horner's syndrome or opiate overdose. Mydriasis is the dilatation of the pupil in response to disease, trauma, drugs (or the dark!) itis pathological when light fails to induce miosis. The radial muscle is innervated by the sympathetic nervous system. Because the parasympathetic fibres travel with the oculomotor nerve they will be damaged by lesions affecting this nerve (e.g. cranial trauma) The response to light shone in one eye is usually a constriction of both pupils. This indicates intact direct and consensual light reflexes. When the optic nerve has an afferent defect the light shining on the affected eye will produce a diminished pupillary response in both eyes. Whereas light shone on the unaffected eye will produce a normal pupillary response in both eyes. This is referred to as the Marcus Gunn pupil and is seen in conditions such as optic neuritis. In a total CN II lesion shining the light in the affected eye will produce no response. ‘The pupillary effects are described above. In addition it supplies all ocular muscles apart from lateral rectus and superior oblique. Thus the affected eye will be deviated inferolaterally. Levator palpebrae superioris may also be impaired resulting in impaired ability to open the eye. ‘The eye will not be able to look down. Largest cranial nerve. Exits the brainstem at the pons. Branches are ophthalmic, maxillary and mandibular. Only the mandibular branch has. both sensory and motor fibres. Branches converge to form the trigeminal ganglion (located in Meckels cave). It supplies the muscles of mastication and also tensor veli palatine, mylohyoid, anterior belly of digastric and tensor tympani. The detailed descriptions of the various sensory functions are described in other areas of the website. The corneal reflex is important and Is elicited by applying a small tip of cotton wool to the cornea, a reflex blink should occur if itis intact. It is, ‘mediated by: the naso ciliary branch of the ophthalmic branch of the trigeminal (sensory component) and the facial nerve producing the ‘motor response. Lesions of the afferent arc will produce bilateral absent blink and lesions of the efferent arc will result in a unilateral absent blink. ‘The affected eye will have a deficit of abduction. This cranial nerve exits the brainstem between the pons and medulla. It thus has a relatively long intra cranial course which renders it susceptible to damage in raised intra cranial pressure. Emerges from brainstem between pons and medulla. it controls muscles of facial expression and taste from the anterior 2/3 of the tongue. The nerve passes into the petrous temporal bone and into the Internal auditory meatus. It then passes through the facial canal and exits at the stylomastoid foramen. It passes through the parotid gland and divides at this point. It does not innervate the parotid gland. Its divisions are considered in other parts of the website. Its motor fibres Vestibulo- cochlear nerve Glossopharyngeal nerve Vagus nerve Accessory nerve Hypoglossal nerve reflex. In surgical practice it may be injured during parotid gland surgery or invaded by malignancies of the gland and a lower motor neurone on the ipsilateral side will result. Exits from the pons and then passes through the internal auditory meatus. It is implicated in sensorineural hearing loss. individuals with sensorineural hearing loss will localise the sound in webers test to the normal ear. Rinnes test will be reduced on the affected side but should still work. These two tests will distinguish sensorineural hearing loss from conductive deafness. In the latter condition webers test will localise to the affected ear and Rinnes test will be impaired on the affected side. Surgical lesions affecting this nerve include CNS tumours and basal skull fractures. It may also be damaged by the administration of ototoxic drugs (of which gentamicin is the most commonly used in surgical practice). Exits the pons just above the vagus. Receives sensory fibres from posterior 1/3 tongue, tonsils, pharynx and middle ear (otalgia may ‘occur following tonsillectomy). It receives visceral afferents from the carotid bodies. It supplies parasympathetic fibres to the parotid gland via the otic ganglion and motor function to stylopharyngeaus muscle. The sensory function of the nerve is tested using the gag reflex. Leaves the medulla between the olivary nucleus and the inferior cerebellar peduncle, Passes through the jugular foramen and into the carotid sheath. Details of the functions of the vagus nerve are covered In the website under relevant organ sub headings. Exists from the caudal aspect of the brainstem (multiple branches) supplies trapezius and sternocleidomastoid muscles. The distal portion of this nerve is most prone to injury during surgical procedures. Emerges from the medulla at the preolivary sulcus, passes through the hypoglossal canal. ities on the carotid sheath and passes deep to the posterior belly of digastric to supply muscles of the tongue (except palatoglossus). Its location near the carotid sheath makes It vulnerable during carotid endarterectomy surgery and damage will produce ipsilateral defect in muscle function Bile Wr Search Tr gy @ © o Question 446 of 611 v Bp oO ‘A 44 year old lady is undergoing an abdominal hysterectomy and the ureter is identified during the ligation of the uterine artery. At which site does it insert into the bladder? Posterior Apex Anterior ‘Superior aspect of the lateral side ‘The ureters enter the bladder at the upper lateral aspect of the base of the bladder. They are about Scm apart from each other in the empty bladder. Internally this aspect is contained within the bladder trigone. Ureter * + 25-35 0m long ‘+ Muscular tube lined by transitional epithelium + Surrounded by thick muscular coat. Becomes 3 muscular layers as it crosses the bony pelvis + Retroperitoneal structure overlying transverse processes L2-L5 ‘+ Lies anterior to bifurcation of iliac vessels + Blood supply is segmental; renal artery, aortic branches, gonadal branches, common iliac and internal iliac + Lies beneath the uterine artery aera Tr By @ © Swen A7 year old boy presents with right iliac fossa pain and there is a clinical suspicion that appendicitis is present. From which of the following embryological structures is the appendix derived? Vitello-intestinal duct Urachus Foregut Hindgut ‘The appendix is derived from the midgut Its derived from the midgut which is why early appendicitis may present with periumbilical pain. wo | | improve Appendix * * Location: Base of caecum. + Upto 10cm long + Mainly lymphoid tissue (Hence mesenteric adenitis may mimic appendicitis). * Caecal taenia coli converge at base of appendix and form a longitudinal muscle cover over, the appendix. This convergence should facilitate its identification at surgery if itis, retrocaecal and difficult to find (which it can be when people start doing appendicectomies!) * Arterial supply: Appendicular artery (branch of the tleocolic) + It is intra peritoneal McBurney's point * 1/3 of the way along a line drawn from the Anterior Superior lliac Spine to the Umbilicus 6 Positions: * Retrocaecal 74% + Pelvic 21% + Postileal * Subcaecal + Paracaecal + Prelleal ee So Question 448 of 611 ’ B oO ‘A surgeon is due to perform a laparotomy for perforated duodenal ulcer. An upper midline incision is to be performed. Which of the following structures is the incision most likely to divide? Rectus abdominis muscle External oblique muscle Internal oblique muscle eo None of the above eo Upper midline abdominal incisions will involve the division of the linea alba. Division of muscles will not usually improve access in this approach and they would not be routinely encountered during this incision. | F | improve re i Abdominal incisions * e * Commonest approach to the abdomen incision ‘= Structures divided: linea alba, transversalis fascia, extraperitoneal fat, peritoneum (avoid falciform ligament above the umbilicus) * Bladder can be accessed via an extraperitoneal approach through the space of Retzius Paramedian * Parallel to the midline (about 3-4cm) incision ‘+ Structures divided/retracted: anterior rectus sheath, rectus (retracted), posterior rectus sheath, transversalis fascia, extraperitoneal fat, peritoneum ‘+ Incision is closed in layers Battle + Similar location to paramedian but rectus displaced medially (and thus denervated) + Now seldom used Kocher's Incision under right subcostal margin e.g. Cholecystectomy (open) Lanz Incision in right iliac fossa e.g. Appendicectomy Gridiron Gable Pfannenstiel's McEvedy's Rutherford Morrison Oblique incision centered over McBurneys point- usually appendicectomy (less cosmetically acceptable than Lanz Rooftop incision Transverse supra pubic, primarily used to access pelvic organs. Groin incision e.g. Emergency repair strangulated femoral hernia Extraperitoneal approach to left or right lower quadrants. Gives excellent access to iliac vessels and is the approach of choice for first time renal transplantation, ‘Abdominal incisions. oc om) Tr gy @ © ‘Save my notes Search Search textbook Q Google search on “Abdominal incisions” + uggest lnk Suggest media °o Question 449 of 611 pe © A 43 year old lady is due to undergo an axillary node clearance as part of treatment for carcinoma of the breast. Which of the following fascial layers will be divided during the surgical approach to the axilla? Sibsons fascia Pre tracheal fascia Waldayers fascia None of the above ‘The clavipectoral fascia is situated under the clavicular portion of pectoralis major. It protects both the axillary vessels and nodes. During an axillary node clearance for breast cancer the clavipectoral fascia Is incised and this allows access to the nodal stations. The nodal stations are; level 1 nodes inferior to pectoralis minor, level 2 lie behind it and level 3 above it. During a Patey Mastectomy surgeons divide pectoralis minor to gain access to level 3 nodes. The use of sentinel node biopsy (and stronger assistants!) have made this procedure far less common. we | @ | Improve Axilla * Boundaries of the axilla Medially Chest wall and Serratus anterior Laterally Humeral head Floor Subscapularis Anterior aspect Lateral border of Pectoralis major Fascia Clavipectoral fascia Content: Long thoracic Derived from C5-C7 and passes behind the brachial plexus to enter the nerve (of Bell) axilla, It lies on the medial chest wall and supplies serratus anterior. Its location puts it at risk during axillary surgery and damage will lead to winging of the scapula. Thoracodorsal nerve and thoracodorsal ‘trunk Axillary vein Intercostobrachial nerves Lymph nodes Search Search textbook Innervate and vascularise latissimus dorsi, Lies at the apex of the axilla it is the continuation of the basilic vein. Becomes the subclavian vein at the outer border of the first rib. Traverse the axillary lymph nodes and are often divided during axillary surgery. They provide cutaneous sensation to the axillary skin. ‘The axilla is the main site of lymphatic drainage for the breast. Q Google search on “Axia” + Suggest ink “suggest media 24 ¢44 B44 4% Dashboard (<) Question 450 of 611 v Bp So ‘Aman undergoes a high anterior resection for carcinoma of the upper rectum. Which of the following vessels will require ligation? Superior mesenteric artery Coeliac axis Perineal artery Middle colic artery The IMA is usually divided during anterior resection. Not only is this borne out of oncological necessity but it also permits sufficient colonic mobilisation for anastomosis. [oot | improve Colon anatomy * The colon commences with the caecum. This represents the most dilated segment of the human colon and its base (which is intraperitoneal) is marked by the convergence of teniae coli. At this, point is located the vermiform appendix. The colon continues as the ascending colon, the posterior aspect of which is retroperitoneal. The line of demarcation between the intra and retro Peritoneal right colon is visible as a white line, in the living, and forms the line of incision for colonic resections. The ascending colon becomes the transverse colon after passing the hepatic flexure. At this location the colon becomes wholly intra peritoneal once again. The superior aspect of the transverse colon is the point of attachment of the transverse colon to the greater omentum. This Is an important anatomical site since division of these attachments permits entry into the lesser sac. Separation of the greater omentum from the transverse colon Is a routine operative step in both gastric and colonic resections. At the left side of the abdomen the transverse colon passes to the left upper quadrant and makes an oblique inferior turn at the splenic flexure, Following this, the posterior aspect becomes retroperitoneal once again. At the level of approximately L4 the descending colon becomes wholly intraperitoneal and becomes the sigmoid colon. Whilst the sigmoid is wholly intraperitoneal there are usually attachments laterally between the sigmoid and the lateral pelvic sidewall. These small congenital adhesions are not formal anatomical attachments but frequently require division during surgical resections, At its distal end the sigmoid passes to the midline and at the region around the sacral promontary it becomes the upper rectum. This transition is visible macroscopically as the point where the teniae fuse. More distally the rectum passes through the peritoneum at the region of the peritoneal reflection and becomes extraperitoneal Arterial supply Superior mesenteric artery and inferior mesenteric artery: inked by the marginal artery Ascending colon: leocolic and right colic arteries. Transverse colon: middle colic artery Descending and sigmoid colon: inferior mesenteric artery Venous drainage From regional veins (that accompany arteries) to superior and inferior mesenteric vein Lymphatic drainage Initially along nodal chains that accompany supplying arteries, then para-aortic nodes Embryology Midgut- Second part of duodenum to 2/3 transverse colon Hindgut- Distal 1/3 transverse colon to anus Peritoneal location The right and left colon are part intraperitoneal and part extraperitoneal. The sigmoid and transverse colon are generally wholly intraperitoneal. This has implications for the sequelae of perforations, which will tend to result In generalised peritonitis in the wholly intra peritoneal segments, Colonic relations Region of colon Relation Caecum/ right colon Right ureter, gonadal vessels Hepatic flexure Gallbladder (medially) Splenic flexure Spleen and tail of pancreas Distal sigmoid/ upper Left ureter rectum Rectum Ureters, autonomic nerves, seminal vesicles, prostate, urethra (aistally) Bie | Save my notes o Question 451 of 611 v B oO ‘An 18 year old man Is stabbed in the axilla during a fight. His axillary artery Is lacerated and repaired. However, the surgeon neglects to repair an associated injury to the upper trunk of the brachial plexus, Which of the following muscles is least likely to demonstrate impaired function as aresult? Infraspinatus Brachialis Supinator brevis None of the above ‘The palmar interossei are supplied by the ulnar nerve. Which lies inferiorly and Is therefore less likely to be injured, [o4 [9 | morove | Ne ibs Brachial plexus * The brachial plexus extends from the neck to the axilla. It is formed by the ventral ramI of the fifth to the eighth cervical nerves with the ascending part of the first thoracic nerve. Location of the plexus ‘The ventral rami which form the plexus enter the lower part of the posterior triangle of the neck in series with the ventral rami of the cervical plexus. The second part of the subclavian artery lies Immediately anterior to the lower two ram|. The upper three ram! intermingle and pass. inferolaterally towards the axilla and subclavian artery. They are enclosed within an extension of the prevertebral fascia. In the neck the plexus lies deep to platysma, the supraciavicular nerves, Inferior belly of omohyoid and the transverse cervical artery. It then passes deep to the clavicle and the suprascapular vessels, to enter the axilla, and thence surround the second part of the axillary artery Composition of the plexus ‘Ventral rami, the roots of the plexus, lie between scalenus medius and anterior. As they enter the posterior triangle, the upper two (C5.6) and lower two (C8, T1) roots of the plexus unite to form the upper and lower trunks of the plexus respectively. Meanwhile, C7 continues as the middle trunk. The lower trunk may groove the superior surface of the first rib posterior to the subclavian artery, and the root from the first ventral ramus is always in contact with it. seesinaieie tamaeiaaieehetareiainaasiiniiaieiaiataadicnniimsatisatetasiieinnianinsthtarintbiesmianmmtimmientens + 4 eiemneaeaneell (flexor) and posterior (extensor) parts of the upper limb. ‘The cords of the plexus are formed in the axilla, The dorsal divisions unite to form the posterior cord (C5-8). The ventral divisions of the upper and middle trunks unite to form the lateral cord (C5-7), while the ventral divisions of the lower trunk continues as the medial cord (C8-T1). The cords are named according to their relationship to the axillary artery. Each cord terminates by dividing into two main branches at the beginning of the third part of the artery. ‘Sympathetic communications The fifth and sixth cervical ventral rami receive grey rami communicantes from the middle cervical ganglion, while the two or more grey rami communicantes pass from the inferior cervical ganglion to the seventh and eighth cervical ventral rami. The first thoracic ventral ramus receives its grey ramus from the cervicothoracic ganglion. Its for this reason that inferior plexus injury can be complicated by a Horners syndrome. summary Origin Anterior rami of C5 to T1 Sections of the + Roots, trunks, divisions, cords, branches plexus ‘+ Mnemonic:Real Teenagers Drink Cold Beer Roots ‘+ Located in the posterior triangle ‘+ Pass between scalenus anterior and medius Trunks ‘+ Located posterior to middle third of clavicle ‘+ Upper and middle trunks related superiorly to the subclavian artery + Lower trunk passes over 1st rib posterior to the subclavian artery Divisions Apex of axilla Cords Related to axillary artery Diagram illustrating the branches of the brachial plexus Divisions, Trunks) Roots Cutaneous sensation of the upper limb son HE tii BileW- ==. ‘Save my notes Search textbook. °e Question 452 of 611 v Bp Oo Which of the following structures separates the subclavian artery and vein? Digastric muscle e Prevertebral fascia Middle scalene muscle oe Omohyoid eo ‘The anterior scalene muscle is an important anatomical landmark and separates the subclavian vein (anterior) from the subclavian artery (posterior). [| o | improve Scalene muscles * The 3 paired muscles are: * Scalenus anterior: Elevate 1st rib and laterally flex the neck to same side + Scalenus medius: Same action as scalenus anterior * Scalenus posterior: Elevate 2nd rib and tilt the cervical spine Innervation Spinalnerves C46 origin Transverse processes C2 10.67 Insertion First and cond rive +The brachial plesus and subclavian artery pass between the anterior and middle scalenes through @ space called the scalenehiatus/fssure aoe ‘+The subclavian vein and phrenic nerve pass anterory to the anterior scalene as It crosses over the first ib, e Q Thoracic outlet syndrome The scalenes are at risk of adhering to the fascia surrounding the brachial plexus or shortening causing compression of the brachial plexus when it passes between the clavicle and 1st rib causing thoracic outlet syndrome. ° cuaeton 453 61 2 = ° What vessel is the origin of the middle rectal artery? Aorta Inferior mesenteric artery Superior mesenteric artery Internal pudendal artery The rectum is supplied by 3 main vessels ‘+ ‘Superior rectal artery from inferior mesenteric artery ‘+ Middle rectal artery from the internal iliac artery * Inferior rectal artery from the internal pudendal artery ir Improve Rectum » ‘The rectum is approximately 12 cm long, It is a capacitance organ. It has both intra and extraperitoneal components. The transition between the sigmoid colon is marked by the disappearance of the tenia coli. The extra peritoneal rectum is surrounded by mesorectal fat that also contains lymph nodes. This mesorectal fatty layer is removed surgically during rectal cancer surgery (Total Mesorectal Excision). The fascial layers that surround the rectum are important clinical landmarks, anteriorly lies the fascia of Denonvilliers. Posteriorly lies Waldeyers fascia, Extra peritoneal rectum + Posterior upper third ‘+ Posterior and lateral middle third ‘+ Whole lower third Relations Anteriorly (Males) Rectovesical pouch Bladder Prostate Seminal vesicles Anteriorly (Females) Posteriorly Laterally Arterial supply Superior rectal artery Venous drainage Superior rectal vein Lymphatic drainage Recto-uterine pouch (Douglas) Cervix Vaginal wall Sacrum Coccyx Middle sacral artery Levator ani Coceygeus ‘+ Mesorectal lymph nodes (superior to dentate line) * Inguinal nodes (inferior to dentate line) ‘Save my notes Search Search textbook Q Google search on "Rectum" suggest lnk ‘suggest media question > Tr & @ © Dashboard

You might also like